Saltar al contenido principal
LibreTexts Español

2.1: Convergencia

  • Page ID
    107810
  • \( \newcommand{\vecs}[1]{\overset { \scriptstyle \rightharpoonup} {\mathbf{#1}} } \)

    \( \newcommand{\vecd}[1]{\overset{-\!-\!\rightharpoonup}{\vphantom{a}\smash {#1}}} \)

    \( \newcommand{\id}{\mathrm{id}}\) \( \newcommand{\Span}{\mathrm{span}}\)

    ( \newcommand{\kernel}{\mathrm{null}\,}\) \( \newcommand{\range}{\mathrm{range}\,}\)

    \( \newcommand{\RealPart}{\mathrm{Re}}\) \( \newcommand{\ImaginaryPart}{\mathrm{Im}}\)

    \( \newcommand{\Argument}{\mathrm{Arg}}\) \( \newcommand{\norm}[1]{\| #1 \|}\)

    \( \newcommand{\inner}[2]{\langle #1, #2 \rangle}\)

    \( \newcommand{\Span}{\mathrm{span}}\)

    \( \newcommand{\id}{\mathrm{id}}\)

    \( \newcommand{\Span}{\mathrm{span}}\)

    \( \newcommand{\kernel}{\mathrm{null}\,}\)

    \( \newcommand{\range}{\mathrm{range}\,}\)

    \( \newcommand{\RealPart}{\mathrm{Re}}\)

    \( \newcommand{\ImaginaryPart}{\mathrm{Im}}\)

    \( \newcommand{\Argument}{\mathrm{Arg}}\)

    \( \newcommand{\norm}[1]{\| #1 \|}\)

    \( \newcommand{\inner}[2]{\langle #1, #2 \rangle}\)

    \( \newcommand{\Span}{\mathrm{span}}\) \( \newcommand{\AA}{\unicode[.8,0]{x212B}}\)

    \( \newcommand{\vectorA}[1]{\vec{#1}}      % arrow\)

    \( \newcommand{\vectorAt}[1]{\vec{\text{#1}}}      % arrow\)

    \( \newcommand{\vectorB}[1]{\overset { \scriptstyle \rightharpoonup} {\mathbf{#1}} } \)

    \( \newcommand{\vectorC}[1]{\textbf{#1}} \)

    \( \newcommand{\vectorD}[1]{\overrightarrow{#1}} \)

    \( \newcommand{\vectorDt}[1]{\overrightarrow{\text{#1}}} \)

    \( \newcommand{\vectE}[1]{\overset{-\!-\!\rightharpoonup}{\vphantom{a}\smash{\mathbf {#1}}}} \)

    \( \newcommand{\vecs}[1]{\overset { \scriptstyle \rightharpoonup} {\mathbf{#1}} } \)

    \( \newcommand{\vecd}[1]{\overset{-\!-\!\rightharpoonup}{\vphantom{a}\smash {#1}}} \)

    Definición\(\PageIndex{1}\)

    \(\left\{a_{n}\right\}\)Sea una secuencia de números reales. Decimos que la secuencia\(\left\{a_{n}\right\}\) converge a\(a \in \mathbb{R}\) si, para alguna\(\varepsilon>0\), existe un entero positivo\(N\) tal que para cualquiera\(n \in \mathbb{N}\) con\(n \geq N\), uno tiene

    \[\left|a_{n}-a\right|<\varepsilon\left(\text { or equivalently }, a-\varepsilon<a_{n}<a+\varepsilon\right).\]

    En este caso, llamamos al límite de\(a\) la secuencia (ver Teorema 2.1.3 a continuación) y escribimos\(\lim _{n \rightarrow \infty} a_{n}=a\). Si la secuencia\(\left\{a_{n}\right\}\) no converge, llamamos a la secuencia divergente.

    Remarcar\(\PageIndex{1}\)

    Se desprende directamente de la definición, utilizando la propiedad de Arquímedes, que una secuencia\(\left\{a_{n}\right\}\) converge a\(a\) si y sólo si por alguna\(\varepsilon>0\), existe un número real\(N\) tal que para cualquiera\(n \in \mathbb{N}\) con\(n>N\), uno tiene

    \[\left|a_{n}-a\right|<\varepsilon. \nonumber\]

    Ejemplo\(\PageIndex{1}\)

    Dejemos\(a_{n}=\frac{1}{n}\) para\(n \in \mathbb{N}\). Eso lo afirmamos\(\lim _{n \rightarrow \infty} a_{n}=0\). Lo verificamos usando la definición.

    Solución

    Vamos\(\varepsilon>0\). Elija un número entero\(N>1 / \varepsilon\). (Tenga en cuenta que dicho número entero\(N\) existe debido a la Propiedad Archimidiana.) Entonces, si\(n \geq N\), obtenemos

    \[\left|a_{n}-0\right|=\left|\frac{1}{n}\right|=\frac{1}{n} \leq \frac{1}{N}<\frac{1}{1 / \varepsilon}=\varepsilon. \nonumber\]

    Ejemplo\(\PageIndex{2}\)

    Ahora generalizamos el ejemplo anterior de la siguiente manera. Consideremos\(\alpha>0\) la secuencia dada por

    \[a_{n}=\frac{1}{n^{\alpha}} \text { for } n \in \mathbb{N}. \nonumber\]

    Solución

    Vamos\(\varepsilon>0\). Elija un entero\(N\) tal que\(N>\left(\frac{1}{\varepsilon}\right)^{1 / \alpha}\). Por cada\(n \geq N\), uno tiene\(n>\left(\frac{1}{\varepsilon}\right)^{1 / \alpha}\) y, por lo tanto,\(n^{\alpha}>\frac{1}{\varepsilon}\). Esto implica

    \[\left|\frac{1}{n^{\alpha}}-0\right|=\frac{1}{n^{\alpha}}<\frac{1}{1 / \varepsilon}=\varepsilon.\]

    Concluimos que\(\lim _{n \rightarrow \infty} a_{n}=0\).

    Ejemplo\(\PageIndex{3}\)

    Considera la secuencia\(\left\{a_{n}\right\}\) donde

    \[a_{n}=\frac{3 n^{2}+4}{2 n^{2}+n+5} \nonumber\]

    Demostraremos directamente a partir de la definición a la que converge esta secuencia\(a= \frac{3}{2}\).

    Solución

    Vamos\(\varepsilon>0\). Primero buscamos un adecuado\(N\). Para ello, simplificamos y estimamos la expresión\(\left|a_{n}-a\right|\). Observe que

    \ [\ begin {alineado}
    \ izquierda|a_ {n} -\ frac {3} {2}\ derecha| &=\ izquierda|\ frac {3 n^ {2} +4} {2 n^ {2} +n+5} -\ frac {3} {2}\ derecha|=\ izquierda|\ frac {2\ izquierda (3 n^ {2} +4\ derecha) -3\ izquierda (2 n^ {2} +n+5\ derecha)} {2\ izquierda (2 n^ {2} +n+5\ derecha)}\ derecha|=\ izquierda|\ frac {-7-3 n} {2\ izquierda (2 n^ {2} +n+5\ derecha)}\ derecha|\\
    &=\ frac {3 n+7} {2 \ izquierda (2 n^ {2} +n+5\ derecha)} <\ frac {10 n} {4 n^ {2}} =\ frac {10} {4 n}
    \ final {alineado}\]

    Para garantizar que la última expresión sea menor que\(\varepsilon\), bastará con elegir\(N>\frac{10}{4 \varepsilon}\). En efecto\(n \geq N\), si, obtenemos

    \[\left|a_{n}-a\right| \leq \frac{10}{4 n} \leq \frac{10}{4 N}<\frac{10}{4 \frac{10}{4 \varepsilon}}=\varepsilon.\]

    Ejemplo\(\PageIndex{4}\)

    \(\left\{a_{n}\right\}\)Déjese dar por

    \[a_{n}=\frac{4 n^{2}-1}{3 n^{2}-n}. \nonumber\]

    Nosotros reclamamos\(\lim _{n \rightarrow \infty} a_{n}=\frac{4}{3}\).

    Solución

    Vamos\(\varepsilon>0\). Buscamos un adecuado\(N\). Primer aviso que

    \[\left|\frac{4 n^{2}-1}{3 n^{2}-n}-\frac{4}{3}\right|=\left|\frac{12 n^{2}-3-12 n^{2}+4 n}{3\left(3 n^{2}-n\right)}\right|=\left|\frac{4 n-3}{3\left(3 n^{2}-n\right)}\right|\]

    Ya que\(n \geq 1\), tenemos\(n^{2} \geq n\) y\(4n>3\). Por lo tanto tenemos

    \[\left|\frac{4 n^{2}-1}{3 n^{2}-n}-\frac{4}{3}\right|=\frac{4 n-3}{3\left(3 n^{2}-n\right)} \leq \frac{4 n-3}{3\left(3 n^{2}-n^{2}\right)}<\frac{4 n}{6 n^{2}}=\frac{4}{6 n}.\]

    Así, si\(N>\frac{4}{6 \varepsilon}\), tenemos, para\(n \geq N\)

    \[\left|\frac{4 n^{2}-1}{3 n^{2}-n}-\frac{4}{3}\right| \leq \frac{4}{6 n} \leq \frac{4}{6 N}<\varepsilon.\]

    Ejemplo\(\PageIndex{5}\)

    Considere la secuencia dada por

    \[a_{n}=\frac{n^{2}+5}{4 n^{2}+n}.\]

    Demostramos directamente a partir de la definición que\(\left\{a_{n}\right\}\) converge a\(\frac{1}{4}\).

    Solución

    Vamos\(\varepsilon>0\). Ahora,

    \[\left|\frac{n^{2}+5}{4 n^{2}+n}-\frac{1}{4}\right|=\left|\frac{4 n^{2}+20-4 n^{2}-n}{4\left(4 n^{2}+n\right)}\right|=\frac{|20-n|}{4\left(4 n^{2}+n\right)}.\]

    Si\(n \geq 20\), entonces\(|20-n|=n-20\). Por lo tanto, para tal\(n\) tenemos

    \[\left|\frac{n^{2}+5}{4 n^{2}+n}-\frac{1}{4}\right|=\frac{n-20}{4\left(4 n^{2}+n\right)} \leq \frac{n}{16 n^{2}}=\frac{1}{16 n}.\]

    Escoge\(N>\max \left\{\frac{1}{16 \varepsilon}, 20\right\}\). Entonces, para\(n \geq N\) nosotros conseguimos

    \[\left|\frac{n^{2}+5}{4 n^{2}+n}-\frac{1}{4}\right| \leq \frac{1}{16 n} \leq \frac{1}{16 N}<\varepsilon.\]

    El siguiente resultado es bastante útil para probar ciertas desigualdades entre números.

    Lema\(\PageIndex{2}\)

    Vamos\(\ell \geq 0\), Si\(\ell<\varepsilon\) por todos\(\varepsilon>0\), entonces\(\ell =0\).

    Prueba

    Esto se demuestra fácilmente por contraposición. Si\(\ell >0\), entonces hay un número positivo, por ejemplo\(\varepsilon=\ell / 2\), tal que\(\varepsilon<\ell\). \(\square\)

    Teorema\(\PageIndex{3}\)

    Una secuencia convergente\(\left\{a_{n}\right\}\) tiene como máximo un límite

    Prueba

    Supongamos que\(\left\{a_{n}\right\}\) converge a\(a\) y\(b\). Entonces dado\(\varepsilon>0\), existen enteros positivos\(N_{1}\) y\(N_{2}\) tales que

    \[\left|a_{n}-a\right|<\varepsilon / 2 \text { for all } n \geq N_{1}\]

    y

    \[\left|a_{n}-b\right|<\varepsilon / 2 \text { for all } n \geq N_{2}.\]

    Vamos\(N=\max \left\{N_{1}, N_{2}\right\}\). Entonces

    \[|a-b| \leq\left|a-a_{N}\right|+\left|a_{N}-b\right|<\varepsilon / 2+\varepsilon / 2=\varepsilon.\]

    Ya que\(\varepsilon>0\) es arbitraria, por Lema 2.1.2,\(|a-b|=0\) y, por lo tanto,\(a=b\). \(\square\)

    El siguiente lema es simple generalización de (2.1.2).

    Lema\(\PageIndex{4}\)

    Dados números reales\(a,b\), entonces\(a \leq b\) si y sólo si\(a<b+\varepsilon\) para todos\(\varepsilon>0\).

    Prueba

    Supongamos\(a<b+\varepsilon\) para todos\(\varepsilon>0\). Y supongamos, a modo de contradicción, eso\(a>b\). luego establecer\(\varepsilon_{0}=a-b\). Entonces\(\varepsilon_{0}>0\). Por suposición, deberíamos haberlo hecho\(a<b+\varepsilon_{0}=b+a-b=a\), lo cual es una contradicción. De ello se deduce que\(a \leq b\).

    La otra dirección sigue inmediatamente de los axiomas de orden. \(\square\)

    El siguiente teorema de comparación muestra que las desigualdades (no estrictas) se conservan “en el límite”.

    Teorema\(\PageIndex{5}\) - Comparison Theorem.

    Supongamos\(\left\{a_{n}\right\}\) y\(\left\{b_{n}\right\}\) convergen hacia\(a\) y\(b\), respectivamente, y\(a_{n} \leq b_{n}\) para todos\(n \in \mathbb{N}\). Entonces\(a \leq b\).

    Prueba

    Para cualquiera\(\varepsilon>0\), existen\(N_{1}, N_{2} \in \mathbb{N}\) tales que

    \[a-\frac{\varepsilon}{2}<a_{n}<a+\frac{\varepsilon}{2}, \quad \text { for } n \geq N_{1},\]

    \[b-\frac{\varepsilon}{2}<b_{n}<b+\frac{\varepsilon}{2}, \quad \text { for } n \geq N_{2}.\]

    Escoge\(N>\max \left\{N_{1}, N_{2}\right\}\). Entonces

    \[a-\frac{\varepsilon}{2}<a_{N} \leq b_{N}<b+\frac{\varepsilon}{2}.\]

    Así,\(a<b+\varepsilon\) para cualquier\(\varepsilon>0\). Usando Lemma 2.1.4 concluimos\(a \leq b\). \(\square\)

    Teorema\(\PageIndex{6}\) - The Squeeze Theorem.

    Supongamos que las secuencias\(\left\{a_{n}\right\}\)\(\left\{b_{n}\right\}\),, y\(\left\{c_{n}\right\}\) satisfacer

    \[a_{n} \leq b_{n} \leq c_{n} \text { for all } n \in \mathbb{N},\]

    y\(\lim _{n \rightarrow \infty} a_{n}=\lim _{n \rightarrow \infty} c_{n}=\ell\). Entonces\(\lim _{n \rightarrow \infty} b_{n}=\ell\).

    Prueba

    Arreglar cualquier\(\varepsilon>0\). Ya que\(\lim _{n \rightarrow \infty} a_{n}=\ell\), existe\(N_{1} \in \mathbb{N}\) tal que

    \[\ell-\varepsilon<a_{n}<\ell+\varepsilon \nonumber\]

    para todos\(n \geq N_{1}\). Del mismo modo\(\displaystyle \lim _{n \rightarrow \infty} c_{n}=\ell\), ya que, existe\(N_{2} \in \mathbb{N}\) tal que

    \[\ell-\varepsilon<c_{n}<\ell+\varepsilon \nonumber\]

    para todos\(n \geq N_{2}\). Vamos\(N=\max \left\{N_{1}, N_{2}\right\}\). Entonces, para\(n \geq N\), tenemos

    \[\ell-\varepsilon<a_{n} \leq b_{n} \leq c_{n}<\ell+\varepsilon, \nonumber\]

    lo que implica\(\left|b_{n}-\ell\right|<\varepsilon\). Por lo tanto,\(\lim _{n \rightarrow \infty} b_{n}=\ell\). \(\square\)

    Definición\(\PageIndex{2}\)

    Una secuencia\(\left\{a_{n}\right\}\) está delimitada arriba si el conjunto\(\left\{a_{n}: n \in \mathbb{N}\right\}\) está delimitado arriba. De igual manera, la secuencia\ left\ {a_ {n}: n\ in\ mathbb {N}\ right\} está delimitada por debajo si el conjunto\(\left\{a_{n}: n \in \mathbb{N}\right\}\) está acotado abajo. Decimos que la secuencia\(\left\{a_{n}\right\}\) está acotada si el conjunto\(\left\{a_{n}: n \in \mathbb{N}\right\}\) está acotado, es decir, si está tanto delimitado arriba como acotado por debajo.

    De la observación posterior a la Definición 1.5.1 se desprende que la secuencia\(\left\{a_{n}\right\}\) está delimitada si y sólo si existe\(M \in \mathbb{R}\) tal que\(\left|a_{n}\right| \leq M\) para todos\(n \in \mathbb{N}\).

    Teorema\(\PageIndex{7}\)

    Una secuencia convergente está delimitada.

    Prueba

    Supongamos que la secuencia\(\left\{a_{n}\right\}\) converge a\(a\). Entonces, pues\(\varepsilon=1\), existe\(N \in \mathbb{N}\) tal que

    \[\left|a_{n}-a\right|<1 \text { for all } n \geq N.\]

    Ya que\(\left|a_{n}\right|-|a| \leq \| a_{n}|-| a|| \leq\left|a_{n}-a\right|\), esto implica\(\left|a_{n}\right|<1+|a|\) para todos\(n \geq N\). Set

    \[M=\max \left\{\left|a_{1}\right|, \ldots,\left|a_{N-1}\right|,|a|+1\right\}\]

    Entonces\(\left|a_{n}\right| \leq M\) para todos\(n \in \mathbb{N}\). Por lo tanto,\(\left\{a_{n}\right\}\) está acotada. \(\square\)

    Definición\(\PageIndex{3}\)

    \(\left\{a_{n}\right\}_{n=1}^{\infty}\)Sea una secuencia de números reales. La secuencia\(\left\{b_{n}\right\}_{n=1}^{\infty}\) se llama una subsecuencia de\(\left\{a_{n}\right\}_{n=1}^{\infty}\) si existe una secuencia de números enteros positivos crecientes

    \[n_{1}<n_{2}<n_{3}<\cdots,\]

    tal que\(b_{k}=a_{n_{k}}\) para cada uno\(k \in \mathbb{N}\).

    Ejemplo\(\PageIndex{6}\)

    Considera la secuencia\(a_{n}=(-1)^{n}\) para\(n \in \mathbb{N}\).

    Solución

    Entonces\(\left\{a_{2 k}\right\}\) es una subsecuencia de\(\left\{a_{n}\right\}\) y\(a_{2 k}=1\) para todos\(k\) (aquí\(n_{k}=2 k\) para todos\(k\)). Del mismo modo,\(\left\{a_{2 k+1}\right\}\) es también una subsecuencia de\(\left\{a_{n}\right\}\) y\(a_{2 k+1}=-1\) para todos\(k\) (aquí\(n_{k}=2 k +1\) para todos\(k\)).

    Lema\(\PageIndex{8}\)

    Let\(\left\{n_{k}\right\}_{k}\) Ser una secuencia de enteros positivos con

    \[n_{1}<n_{2}<n_{3}<\cdots\]

    Entonces\(n_{k} \geq k\) para todos\(k \in \mathbb{N}\).

    Prueba

    Utilizamos la inducción matemática. Cuando\(k=1\), es claro que\(n_{1} \geq 1\) ya\(n_{1}\) es un entero positivo. Asumir\(n_{k} \geq k\) para algunos\(k\). Ahora\(n_{k+1}>n_{k}\) y, ya que\(n_{k}\) y\(n_{k+1}\) son enteros, esto implica,\(n_{k+1} \geq n_{k}+1\). Por lo tanto,\(n_{k+1} \geq k+1\) por la hipótesis inductiva. La conclusión sigue ahora por el principio de inducción matemática. \(\square\)

    Teorema\(\PageIndex{9}\)

    Si una secuencia\(\left\{a_{n}\right\}\) converge a\(a\), entonces cualquier subsecuencia\(\left\{a_{n_{k}}\right\}\) de\(\left\{a_{n}\right\}\) también converge a\(a\).

    Prueba

    Supongamos que\(\left\{a_{n}\right\}\) converge\(a\) y deja que\(\varepsilon>0\) se le dé. Entonces existe\(N\) tal que

    \[\left|a_{n}-a\right|<\varepsilon \text { for all } n \geq N.\]

    Para cualquiera\(k \geq N\), ya que\(n_{k} \geq k\), también tenemos

    \[\left|a_{n_{k}}-a\right|<\varepsilon.\]

    Así,\(\left\{a_{n_{k}}\right\}\) converge a\(a\) como\(k \rightarrow \infty\). \(\square\)

    Ejemplo\(\PageIndex{1}\)

    Dejemos\(a_{n}=(-1)^{n}\) para\(n \in \mathbb{N}\).

    Solución

    Entonces la secuencia\(\left\{a_{n}\right\}\) es divergente. En efecto supongamos por contradicción que

    \(\lim _{n \rightarrow \infty} a_{n}=\ell\).

    Entonces cada subsecuencia de\(\left\{a_{n}\right\}\) converge a un número\(\ell \in \mathbb{R}\). Del teorema anterior se deduce, en particular, que

    \(\ell=\lim _{k \rightarrow \infty} a_{2 k}=1 \text { and } \ell=\lim _{k \rightarrow \infty} a_{2 k+1}=-1\).

    Esta contradicción demuestra que la secuencia es divergente.

    Dado que la secuencia\(\left\{a_{n}\right\}\) es acotada pero no convergente, este ejemplo ilustra el hecho de que lo contrario del teorema 2.1.7 no es cierto.

    Remarcar\(\PageIndex{10}\)

    Dado un entero positivo\(k_{0}\), será conveniente hablar también de la secuencia\(\left\{a_{n}\right\}_{n \geq k_{0}}\), es decir, una función definida solo para los enteros mayores o iguales a\(k_{0}\). Para simplificar la notación, también podemos denotar esta secuencia por\(\left\{a_{n}\right\}\) cada vez que el entero\(k_{0}\) es claro desde el contexto. Por ejemplo, hablamos de la secuencia\(\left\{a_{n}\right\}\) dada por

    \[a_{n}=\frac{n+1}{(n-1)(n-2)}.\]

    aunque\(a_{1}\) y no\(a_{2}\) están definidos. En todos los casos, la secuencia debe definirse a partir de algún entero en adelante.

    Ejercicio\(\PageIndex{1}\)

    Demostrar lo siguiente directamente a partir de la definición de límite.

    1. \(\lim _{n \rightarrow \infty} \frac{2 n^{2}+2}{3 n^{3}+1}=0\).
    2. \(\lim _{n \rightarrow \infty} \frac{n^{2}+1}{5 n^{2}+n+1}=\frac{1}{5}\).
    3. \(\lim _{n \rightarrow \infty} \frac{2 n^{3}+1}{4 n^{3}-n}=\frac{1}{2}\).
    4. \(\lim _{n \rightarrow \infty} \frac{3 n^{2}+5}{6 n^{2}+n}=\frac{1}{2}\).
    5. \(\lim _{n \rightarrow \infty} \frac{4 n^{2}-1}{n^{2}-n}=4\).
    Contestar

    Agrega textos aquí. No borre primero este texto.

    Ejercicio\(\PageIndex{2}\)

    Demostrar que si\(\left\{a_{n}\right\}\) es una secuencia convergente, entonces\(\left\{|a_{n}|\right\}\) es una secuencia convergente. ¿Es cierto lo contrario?

    Contestar

    Agrega textos aquí. No borre primero este texto.

    Ejercicio\(\PageIndex{3}\)

    Dejar\(\left\{a_{n}\right\}\) ser una secuencia. Demostrar que si el\(\left\{|a_{n}|\right\}\) converge a 0, entonces\(\left\{a_{n}\right\}\) también converge a 0.

    Contestar

    Agrega textos aquí. No borre primero este texto.

    Ejercicio\(\PageIndex{4}\)

    \(\lim _{n \rightarrow \infty} \frac{\sin n}{n}=0\)Demuéstralo.

    Contestar

    Agrega textos aquí. No borre primero este texto.

    Ejercicio\(\PageIndex{5}\)

    Dejar\(\left\{x_{n}\right\}\) ser una secuencia acotada y dejar\(\left\{y_{n}\right\}\) ser una secuencia que converja a 0. Demostrar que la secuencia\(\left\{x_{n} y_{n}\right\}\) converge a 0.

    Contestar

    Agrega textos aquí. No borre primero este texto.

    Ejercicio\(\PageIndex{6}\)

    Demostrar que los siguientes límites son 0. (Pista: use Teorema 2.1.6.)

    1. \(\lim _{n \rightarrow \infty} \frac{n+\cos \left(n^{2}-3\right)}{2 n^{2}+1}\).
    2. \(\lim _{n \rightarrow \infty} \frac{3^{n}}{n !}\).
    3. \(\lim _{n \rightarrow \infty} \frac{n !}{n^{n}}\).
    4. \(\lim _{n \rightarrow \infty} \frac{n^{2}}{3^{n}}\). (Pista: ver Ejercicio 1.3.4 (c)).
    Contestar

    Agrega textos aquí. No borre primero este texto.

    Ejercicio\(\PageIndex{7}\)

    Demostrar que si\(\lim _{n \rightarrow \infty} a_{n}=\ell>0\), entonces existe\(N \in \mathbb{N}\) tal que\(a_{n}>0\) para todos\(n \geq N\).

    Contestar

    Agrega textos aquí. No borre primero este texto.

    Ejercicio\(\PageIndex{8}\)

    Demostrar que si\(\lim _{n \rightarrow \infty} a_{n}=\ell \neq 0\), entonces\(\lim _{n \rightarrow \infty} \frac{a_{n+1}}{a_{n}}=1\). ¿Sigue siendo cierta la conclusión si\(\ell =0\)?

    Contestar

    Agrega textos aquí. No borre primero este texto.

    Ejercicio\(\PageIndex{9}\)

    Dejemos\(\left\{a_{n}\right\}\) ser una secuencia de números reales tales que\(\lim _{n \rightarrow \infty} a_{n}=3\). Utilice la Definición 2.1.1 para probar lo siguiente

    1. \(\lim _{n \rightarrow \infty} 3 a_{n}-7=2\);
    2. \(\lim _{n \rightarrow \infty} \frac{a_{n}+1}{a_{n}}=\frac{4}{3}\); (Pista: probar primero que hay\(N\) tal que\(a_{n}>1\) para\(n \geq N\).)
    Contestar

    Agrega textos aquí. No borre primero este texto.

    Ejercicio\(\PageIndex{10}\)

    Dejemos\(a_{n} \geq 0\) para todos\(n \in \mathbb{N}\). Demostrar que si\(\lim _{n \rightarrow \infty} a_{n}=\ell\), entonces\(\lim _{n \rightarrow \infty} \sqrt{a_{n}}=\sqrt{\ell}\).

    Contestar

    Agrega textos aquí. No borre primero este texto.

    Ejercicio\(\PageIndex{11}\)

    Demostrar que la secuencia\(\left\{a_{n}\right\}\) con\(a_{n}=\sin (n \pi / 2)\) es divergente.

    Contestar

    Agrega textos aquí. No borre primero este texto.

    Ejercicio\(\PageIndex{12}\)

    Considera una secuencia\(\left\{a_{n}\right\}\).

    1. Demostrar que\(\lim _{n \rightarrow \infty} a_{n}=\ell\) si y sólo si\(\lim _{k \rightarrow \infty} a_{2 k}=\ell\) y\(\lim _{k \rightarrow \infty} a_{2 k+1}=\ell\).
    2. Demostrar que\(\lim _{n \rightarrow \infty} a_{n}=\ell\) si y sólo si\(\lim _{k \rightarrow \infty} a_{3 k}=\ell\),\(\lim _{k \rightarrow \infty} a_{3 k+1}=\ell\), y\(\lim _{k \rightarrow \infty} a_{3 k+2}=\ell\).
    Contestar

    Agrega textos aquí. No borre primero este texto.

    Ejercicio\(\PageIndex{13}\)

    Dada una secuencia\(\left\{a_{n}\right\}\), definir una nueva secuencia\(\left\{b_{n}\right\}\) mediante

    \[b_{n}=\frac{a_{1}+a_{2}+\ldots+a_{n}}{n}.\]

    1. Demostrar que si\(\lim _{n \rightarrow \infty} a_{n}=\ell\), entonces\(\lim _{n \rightarrow \infty} b_{n}=\ell\)
    2. Encuentra un contraejemplo para demostrar que lo contrario no se sostiene en general.
    Contestar

    Agrega textos aquí. No borre primero este texto.


    This page titled 2.1: Convergencia is shared under a CC BY-NC-SA license and was authored, remixed, and/or curated by Lafferriere, Lafferriere, and Nguyen (PDXOpen: Open Educational Resources) .